Difference between revisions of "Aufgaben:Exercise 4.3: Different Frequencies"

From LNTwww
Line 1: Line 1:
  
{{quiz-Header|Buchseite=Digitalsignalübertragung/Signale, Basisfunktionen und Vektorräume}}
+
{{quiz-Header|Buchseite=Digital_Signal_Transmission/Signals,_Basis_Functions_and_Vector_Spaces}}
  
[[File:P_ID1999__Dig_A_4_3.png|right|frame|Vorgegebene Signalmenge  $\{s_i(t)\}$]]
+
[[File:P_ID1999__Dig_A_4_3.png|right|frame|Given signal set  $\{s_i(t)\}$]]
In der Grafik sind  $M = 5$  verschiedene Signale  $s_i(t)$  dargestellt. Entgegen der Nomenklatur im Theorieteil sind für die Laufvariable  $i$  die Werte  $0, \ \text{...} \ , M-1$  möglich.
+
In the diagram  $M = 5$  different signals  $s_i(t)$  are shown. Contrary to the nomenclature in the theory section, the indexing variable  $i$  can have the values  $0, \ \text{...} \ , M-1$.   
  
Anzumerken ist:
+
To be noted:
* Alle Signale sind zeitbegrenzt auf  $0$  bis  $T$; damit ist auch die Energie aller Signale endlich.
+
* All signals are time-limited to  $0$  to  $T$; thus the energy of all signals is also finite.
* Das Signal  $s_1(t)$  hat die Periodendauer  $T_0 = T$. Die Frequenz ist damit gleich  $f_0 = 1/T$.
+
* The signal  $s_1(t)$  has the period  $T_0 = T$. The frequency is therefore equal to  $f_0 = 1/T$.
* Die Signale  $s_i(t)$  mit  $i ≠ 0$  sind Cosinusschwingungen mit der Frequenz  $i \cdot f_0$.  
+
* The signals  $s_i(t)$  with  $i ≠ 0$  are cosine oscillations with frequency  $i \cdot f_0$.  
*Dagegen ist  $s_0(t)$  zwischen  $0$  und  $T$  konstant.
+
*In contrast,  $s_0(t)$  is constant between  $0$  and  $T$.   
* Der Maximalwert aller Signale ist  $A$  und es gilt  $|s_i(t)| ≤ A$.
+
* The maximum value of all signals is  $A$  and  $|s_i(t)| ≤ A$ holds.
  
  
Gesucht sind in dieser Aufgabe die  $N$  Basisfunktionen, die hier entgegen der bisherigen Beschreibung im Theorieteil mit  $j = 0, \ \text{...} \ , N-1$  durchnummeriert werden.
+
In this exercise we are looking for the  $N$  basis functions, which are numbered here with  $j = 0, \ \text{...} \ , N-1$. 
  
  
Line 19: Line 19:
  
  
''Hinweis:''  
+
''Note:''  
*Die Aufgabe gehört zum  Kapitel   [[Digitalsignal%C3%BCbertragung/Signale,_Basisfunktionen_und_Vektorr%C3%A4ume| Signale, Basisfunktionen und Vektorräume]].
+
*The exercise belongs to the chapter   [[Digital_Signal_Transmission/Signals,_Basis_Functions_and_Vector_Spaces|"Signals, Basis Functions and Vector Spaces"]].
 
   
 
   
  
  
  
===Fragebogen===
+
===Question===
 
<quiz display=simple>
 
<quiz display=simple>
{Beschreiben Sie die Signalmenge&nbsp; $\{s_i(t)\}$&nbsp; mit&nbsp;  $0 &#8804; i &#8804; 4$&nbsp; möglichst kompakt. <br>Welche Beschreibungsform ist richtig?
+
{Describe the signal set&nbsp; $\{s_i(t)\}$&nbsp; with&nbsp;  $0 &#8804; i &#8804; 4$&nbsp; as compactly as possible. <br>Which description form is correct?
 
|type="[]"}
 
|type="[]"}
 
- $s_i(t) = A \cdot \cos {(2\pi \cdot i \cdot t/T)}$.
 
- $s_i(t) = A \cdot \cos {(2\pi \cdot i \cdot t/T)}$.
+ $s_i(t) = A \cdot \cos {(2\pi \cdot i \cdot t/T)}$&nbsp; für &nbsp;$0 &#8804; t < T$, &nbsp;sonst $0$.
+
+ $s_i(t) = A \cdot \cos {(2\pi \cdot i \cdot t/T)}$&nbsp; for &nbsp;$0 &#8804; t < T$, &nbsp;otherwise $0$.
- $s_i(t) = A \cdot \cos {(2\pi t/T \, &ndash; \, i \cdot \pi/2)}$&nbsp; für &nbsp;$0 &#8804; t < T$, &nbsp;sonst $0$.
+
- $s_i(t) = A \cdot \cos {(2\pi t/T \, &ndash; \, i \cdot \pi/2)}$&nbsp; for &nbsp;$0 &#8804; t < T$, &nbsp;otherwise $0$.
  
{Geben Sie die Anzahl&nbsp; $N$&nbsp; der erforderlichen Basisfunktionen an.
+
{Specify the number&nbsp; $N$&nbsp; of basis functions required.
 
|type="{}"}
 
|type="{}"}
 
$N \ = \ $ { 5 3% }
 
$N \ = \ $ { 5 3% }
  
{Wie lautet die Basisfunktion&nbsp; $\varphi_0(t)$, die formgleich mit&nbsp; $s_0(t)$&nbsp; ist?
+
{What is the basis function&nbsp; $\varphi_0(t)$ that is equal in form to&nbsp; $s_0(t)$?&nbsp;  
 
|type="[]"}
 
|type="[]"}
 
- $\varphi_0(t) = s_0(t)$,
 
- $\varphi_0(t) = s_0(t)$,
+ $\varphi_0(t) = \sqrt{1/T}$ für $0 &#8804; t < T$, &nbsp;außerhalb &nbsp;$0$.
+
+ $\varphi_0(t) = \sqrt{1/T}$ for $0 &#8804; t < T$, &nbsp;outside &nbsp;$0$.
- $\varphi_0(t) = \sqrt{2/T}$ für $0 &#8804; t < T$, &nbsp;außerhalb &nbsp;$0$.
+
- $\varphi_0(t) = \sqrt{2/T}$ for $0 &#8804; t < T$, &nbsp;outside &nbsp;$0$.
  
{Wie lautet die Basisfunktion&nbsp; $\varphi_1(t)$, die formgleich mit&nbsp;  $s_1(t)$&nbsp; ist?
+
{What is the basis function&nbsp; $\varphi_1(t)$ that is equal in form to&nbsp;  $s_1(t)$?&nbsp;  
 
|type="[]"}
 
|type="[]"}
 
- $\varphi_1(t) = s_1(t)$,
 
- $\varphi_1(t) = s_1(t)$,
- $\varphi_1(t) = \sqrt{1/T} \cdot \cos {(2\pi t/T)}$ für $0 &#8804; t < T$, &nbsp;außerhalb $0$.
+
- $\varphi_1(t) = \sqrt{1/T} \cdot \cos {(2\pi t/T)}$ for $0 &#8804; t < T$, &nbsp;outside $0$.
+ $\varphi_1(t) =\sqrt{2/T} \cdot \cos {(2\pi t/T)}$ für $0 &#8804; t < T$, &nbsp;außerhalb $0$.
+
+ $\varphi_1(t) =\sqrt{2/T} \cdot \cos {(2\pi t/T)}$ for $0 &#8804; t < T$, &nbsp;outside $0$.
 
</quiz>
 
</quiz>
  
===Musterlösung===
+
===Solution===
 
{{ML-Kopf}}
 
{{ML-Kopf}}
'''(1)'''&nbsp; Richtig ist der <u>Lösungsvorchlag 2</u>:
+
'''(1)'''&nbsp; Correct is the <u>solution 2</u>:
* Dieser berücksichtigt die unterschiedlichen Frequenzen und die Begrenzung auf den Bereich $0 &#8804; t < T$.  
+
* This takes into account the different frequencies and the limitation to the range $0 &#8804; t < T$.  
*Die Signale $s_i(t)$ gemäß Vorschlag 3 unterscheiden sich dagegen nicht bezüglich der Frequenz, sondern weisen unterschiedliche Phasenlagen auf.
+
*The signals $s_i(t)$ according to suggestion 3, on the other hand, do not differ with respect to frequency, but have different phase positions.
  
  
  
'''(2)'''&nbsp; Die energiebegrenzten Signale $s_i(t) = A \cdot \cos {(2\pi \cdot i \cdot t/T)}$ sind alle zueinander orthogonal, das heißt, dass das innere Produkt zweier Signale $s_i(t)$ und $s_k(t)$ mit $i &ne; k$ stets $0$ ist :
+
'''(2)'''&nbsp; The energy-limited signals $s_i(t) = A \cdot \cos {(2\pi \cdot i \cdot t/T)}$ are all orthogonal to each other, that is, the inner product of two signals $s_i(t)$ and $s_k(t)$ with $i &ne; k$ is always $0$:
 
:$$< \hspace{-0.1cm}s_i(t), \hspace{0.1cm} s_k(t)\hspace{-0.1cm} > \hspace{0.1cm} \hspace{-0.1cm} \ = \ \hspace{-0.1cm} A^2 \cdot \int_{0}^{T}\cos(2\pi \cdot i \cdot t/T) \cdot \cos(2\pi \cdot k \cdot t/T)\,{\rm d} t $$
 
:$$< \hspace{-0.1cm}s_i(t), \hspace{0.1cm} s_k(t)\hspace{-0.1cm} > \hspace{0.1cm} \hspace{-0.1cm} \ = \ \hspace{-0.1cm} A^2 \cdot \int_{0}^{T}\cos(2\pi \cdot i \cdot t/T) \cdot \cos(2\pi \cdot k \cdot t/T)\,{\rm d} t $$
 
:$$ \Rightarrow \hspace{0.3cm} < \hspace{-0.1cm}s_i(t), \hspace{0.1cm} s_k(t)\hspace{-0.1cm} > \hspace{0.1cm} \hspace{-0.1cm}  {A^2}/{2} \cdot \int_{0}^{T}\cos(2\pi (i-k) t/T) \,{\rm d} t +
 
:$$ \Rightarrow \hspace{0.3cm} < \hspace{-0.1cm}s_i(t), \hspace{0.1cm} s_k(t)\hspace{-0.1cm} > \hspace{0.1cm} \hspace{-0.1cm}  {A^2}/{2} \cdot \int_{0}^{T}\cos(2\pi (i-k) t/T) \,{\rm d} t +
Line 64: Line 64:
 
   \hspace{0.05cm}.$$
 
   \hspace{0.05cm}.$$
  
*Mit $i &#8712; \{0, \ \text{...} \ , 4\}$ und $k &#8712; \{0, \ \text{...}\ , 4\}$ sowie $i &ne; j$ ist sowohl $i \, &ndash; k$ ganzzahlig ungleich $0$, ebenso die Summe $i + k$.  
+
*With $i &#8712; \{0, \ \text{...} \ , 4\}$ and $k &#8712; \{0, \ \text{...}\ , 4\}$ as well as $i &ne; j$, both $i \, &ndash; k$ is integer not equal to $0$, as is the sum $i + k$.  
*Dadurch liefern beide Integrale das Ergebnis Null:
+
*Thus, both integrals yield the result zero:
 
:$$< \hspace{-0.1cm}s_i(t), \hspace{0.1cm} s_k(t)\hspace{-0.1cm} > \hspace{0.1cm} \hspace{-0.1cm}= 0  
 
:$$< \hspace{-0.1cm}s_i(t), \hspace{0.1cm} s_k(t)\hspace{-0.1cm} > \hspace{0.1cm} \hspace{-0.1cm}= 0  
 
  \hspace{0.3cm}\Rightarrow \hspace{0.3cm}  \hspace{0.05cm}\hspace{0.15cm}\underline {N = M = 5}
 
  \hspace{0.3cm}\Rightarrow \hspace{0.3cm}  \hspace{0.05cm}\hspace{0.15cm}\underline {N = M = 5}
Line 71: Line 71:
  
  
'''(3)'''&nbsp; Die Energie des innerhalb $T$ konstanten Signals $s_0(t)$ ist gleich
+
'''(3)'''&nbsp; The energy of the signal $s_0(t)$, which is constant within $T$, is equal to
 
:$$E_0 = ||s_0(t)||^2 = A^2 \cdot T  
 
:$$E_0 = ||s_0(t)||^2 = A^2 \cdot T  
 
  \hspace{0.3cm}\Rightarrow \hspace{0.3cm} ||s_0(t)|| = A \cdot \sqrt{T}  \hspace{0.3cm}  
 
  \hspace{0.3cm}\Rightarrow \hspace{0.3cm} ||s_0(t)|| = A \cdot \sqrt{T}  \hspace{0.3cm}  
Line 78: Line 78:
 
  0  \end{array} \right.\quad
 
  0  \end{array} \right.\quad
 
\begin{array}{*{1}c} 0 \le t < T \hspace{0.05cm},
 
\begin{array}{*{1}c} 0 \le t < T \hspace{0.05cm},
\\  {\rm sonst}\hspace{0.05cm}. \\ \end{array}$$
+
\\  {\rm otherwise}\hspace{0.05cm}. \\ \end{array}$$
  
Richtig ist demzufolge der <u>Lösungsvorschlag 2</u>.
+
Therefore, <u>solution 2</u> is correct.
  
  
'''(4)'''&nbsp; Richtig ist hier der <u>letzte Lösungsvorschlag</u> wegen
+
'''(4)'''&nbsp; The <u>last solution</u> is correct because of
 
:$$E_1 = ||s_1(t)||^2 = \frac{A^2 \cdot T}{2}  
 
:$$E_1 = ||s_1(t)||^2 = \frac{A^2 \cdot T}{2}  
 
  \hspace{0.3cm}\Rightarrow \hspace{0.3cm} ||s_1(t)|| = A \cdot \sqrt{{T}/{2}} \hspace{0.3cm}  
 
  \hspace{0.3cm}\Rightarrow \hspace{0.3cm} ||s_1(t)|| = A \cdot \sqrt{{T}/{2}} \hspace{0.3cm}  
Line 90: Line 90:
 
  0  \end{array} \right.\quad
 
  0  \end{array} \right.\quad
 
\begin{array}{*{1}c} 0 \le t < T \hspace{0.05cm},
 
\begin{array}{*{1}c} 0 \le t < T \hspace{0.05cm},
\\  {\rm sonst}\hspace{0.05cm}. \\ \end{array}$$
+
\\  {\rm otherwise}\hspace{0.05cm}. \\ \end{array}$$
 
{{ML-Fuß}}
 
{{ML-Fuß}}
  

Revision as of 22:43, 14 June 2022

Given signal set  $\{s_i(t)\}$

In the diagram  $M = 5$  different signals  $s_i(t)$  are shown. Contrary to the nomenclature in the theory section, the indexing variable  $i$  can have the values  $0, \ \text{...} \ , M-1$. 

To be noted:

  • All signals are time-limited to  $0$  to  $T$; thus the energy of all signals is also finite.
  • The signal  $s_1(t)$  has the period  $T_0 = T$. The frequency is therefore equal to  $f_0 = 1/T$.
  • The signals  $s_i(t)$  with  $i ≠ 0$  are cosine oscillations with frequency  $i \cdot f_0$.
  • In contrast,  $s_0(t)$  is constant between  $0$  and  $T$. 
  • The maximum value of all signals is  $A$  and  $|s_i(t)| ≤ A$ holds.


In this exercise we are looking for the  $N$  basis functions, which are numbered here with  $j = 0, \ \text{...} \ , N-1$. 



Note:



Question

1

Describe the signal set  $\{s_i(t)\}$  with  $0 ≤ i ≤ 4$  as compactly as possible.
Which description form is correct?

$s_i(t) = A \cdot \cos {(2\pi \cdot i \cdot t/T)}$.
$s_i(t) = A \cdot \cos {(2\pi \cdot i \cdot t/T)}$  for  $0 ≤ t < T$,  otherwise $0$.
$s_i(t) = A \cdot \cos {(2\pi t/T \, – \, i \cdot \pi/2)}$  for  $0 ≤ t < T$,  otherwise $0$.

2

Specify the number  $N$  of basis functions required.

$N \ = \ $

3

What is the basis function  $\varphi_0(t)$ that is equal in form to  $s_0(t)$? 

$\varphi_0(t) = s_0(t)$,
$\varphi_0(t) = \sqrt{1/T}$ for $0 ≤ t < T$,  outside  $0$.
$\varphi_0(t) = \sqrt{2/T}$ for $0 ≤ t < T$,  outside  $0$.

4

What is the basis function  $\varphi_1(t)$ that is equal in form to  $s_1(t)$? 

$\varphi_1(t) = s_1(t)$,
$\varphi_1(t) = \sqrt{1/T} \cdot \cos {(2\pi t/T)}$ for $0 ≤ t < T$,  outside $0$.
$\varphi_1(t) =\sqrt{2/T} \cdot \cos {(2\pi t/T)}$ for $0 ≤ t < T$,  outside $0$.


Solution

(1)  Correct is the solution 2:

  • This takes into account the different frequencies and the limitation to the range $0 ≤ t < T$.
  • The signals $s_i(t)$ according to suggestion 3, on the other hand, do not differ with respect to frequency, but have different phase positions.


(2)  The energy-limited signals $s_i(t) = A \cdot \cos {(2\pi \cdot i \cdot t/T)}$ are all orthogonal to each other, that is, the inner product of two signals $s_i(t)$ and $s_k(t)$ with $i ≠ k$ is always $0$:

$$< \hspace{-0.1cm}s_i(t), \hspace{0.1cm} s_k(t)\hspace{-0.1cm} > \hspace{0.1cm} \hspace{-0.1cm} \ = \ \hspace{-0.1cm} A^2 \cdot \int_{0}^{T}\cos(2\pi \cdot i \cdot t/T) \cdot \cos(2\pi \cdot k \cdot t/T)\,{\rm d} t $$
$$ \Rightarrow \hspace{0.3cm} < \hspace{-0.1cm}s_i(t), \hspace{0.1cm} s_k(t)\hspace{-0.1cm} > \hspace{0.1cm} \hspace{-0.1cm} {A^2}/{2} \cdot \int_{0}^{T}\cos(2\pi (i-k) t/T) \,{\rm d} t + \frac{A^2}{2} \cdot \int_{0}^{T}\cos(2\pi (i+k) t/T) \,{\rm d} t \hspace{0.05cm}.$$
  • With $i ∈ \{0, \ \text{...} \ , 4\}$ and $k ∈ \{0, \ \text{...}\ , 4\}$ as well as $i ≠ j$, both $i \, – k$ is integer not equal to $0$, as is the sum $i + k$.
  • Thus, both integrals yield the result zero:
$$< \hspace{-0.1cm}s_i(t), \hspace{0.1cm} s_k(t)\hspace{-0.1cm} > \hspace{0.1cm} \hspace{-0.1cm}= 0 \hspace{0.3cm}\Rightarrow \hspace{0.3cm} \hspace{0.05cm}\hspace{0.15cm}\underline {N = M = 5} \hspace{0.05cm}.$$


(3)  The energy of the signal $s_0(t)$, which is constant within $T$, is equal to

$$E_0 = ||s_0(t)||^2 = A^2 \cdot T \hspace{0.3cm}\Rightarrow \hspace{0.3cm} ||s_0(t)|| = A \cdot \sqrt{T} \hspace{0.3cm} \Rightarrow \hspace{0.3cm} \varphi_0 (t) = \frac{s_0(t)}{||s_0(t)||} = \left\{ \begin{array}{c} 1/\sqrt{T} \\ 0 \end{array} \right.\quad \begin{array}{*{1}c} 0 \le t < T \hspace{0.05cm}, \\ {\rm otherwise}\hspace{0.05cm}. \\ \end{array}$$

Therefore, solution 2 is correct.


(4)  The last solution is correct because of

$$E_1 = ||s_1(t)||^2 = \frac{A^2 \cdot T}{2} \hspace{0.3cm}\Rightarrow \hspace{0.3cm} ||s_1(t)|| = A \cdot \sqrt{{T}/{2}} \hspace{0.3cm} \Rightarrow \hspace{0.3cm} \varphi_1 (t) = \frac{s_1(t)}{||s_1(t)||} = \left\{ \begin{array}{c} \sqrt{2/T} \cdot \cos(2\pi t/T) \\ 0 \end{array} \right.\quad \begin{array}{*{1}c} 0 \le t < T \hspace{0.05cm}, \\ {\rm otherwise}\hspace{0.05cm}. \\ \end{array}$$